$int _0^infty f(x) $ exists and $f(x)$ is differentiable then $lim _{x to infty} f '(x)$ exists. Counter...












7














Can anyone give me a counter example of the statement




If $int_0^infty f(x) $ exists and $f(x)$ is differentiable then $lim _{x to infty} f'(x)$ exists.




My attempt: I have thought one. First I draw $1/x^2$ in the first quadrant and $-1/x^2$ in the fourth quadrant. The area under the following curves are finite.



1) $1/x^2$



2) $-1/x^2$



3) $x= 1$.



Now I have drawn infinite number of $y = x+c $ at equal distances in that region. Then I joined those infinite lines by some smooth curve so that the curve remains differentiable. Now I think this function can be a counter example.



I am uploading one picture of my attempt. Can anyone please check it and if possible suggest me a better function.enter image description here










share|cite|improve this question





























    7














    Can anyone give me a counter example of the statement




    If $int_0^infty f(x) $ exists and $f(x)$ is differentiable then $lim _{x to infty} f'(x)$ exists.




    My attempt: I have thought one. First I draw $1/x^2$ in the first quadrant and $-1/x^2$ in the fourth quadrant. The area under the following curves are finite.



    1) $1/x^2$



    2) $-1/x^2$



    3) $x= 1$.



    Now I have drawn infinite number of $y = x+c $ at equal distances in that region. Then I joined those infinite lines by some smooth curve so that the curve remains differentiable. Now I think this function can be a counter example.



    I am uploading one picture of my attempt. Can anyone please check it and if possible suggest me a better function.enter image description here










    share|cite|improve this question



























      7












      7








      7


      3





      Can anyone give me a counter example of the statement




      If $int_0^infty f(x) $ exists and $f(x)$ is differentiable then $lim _{x to infty} f'(x)$ exists.




      My attempt: I have thought one. First I draw $1/x^2$ in the first quadrant and $-1/x^2$ in the fourth quadrant. The area under the following curves are finite.



      1) $1/x^2$



      2) $-1/x^2$



      3) $x= 1$.



      Now I have drawn infinite number of $y = x+c $ at equal distances in that region. Then I joined those infinite lines by some smooth curve so that the curve remains differentiable. Now I think this function can be a counter example.



      I am uploading one picture of my attempt. Can anyone please check it and if possible suggest me a better function.enter image description here










      share|cite|improve this question















      Can anyone give me a counter example of the statement




      If $int_0^infty f(x) $ exists and $f(x)$ is differentiable then $lim _{x to infty} f'(x)$ exists.




      My attempt: I have thought one. First I draw $1/x^2$ in the first quadrant and $-1/x^2$ in the fourth quadrant. The area under the following curves are finite.



      1) $1/x^2$



      2) $-1/x^2$



      3) $x= 1$.



      Now I have drawn infinite number of $y = x+c $ at equal distances in that region. Then I joined those infinite lines by some smooth curve so that the curve remains differentiable. Now I think this function can be a counter example.



      I am uploading one picture of my attempt. Can anyone please check it and if possible suggest me a better function.enter image description here







      limits analysis derivatives proof-verification continuity






      share|cite|improve this question















      share|cite|improve this question













      share|cite|improve this question




      share|cite|improve this question








      edited Nov 22 at 5:07









      gt6989b

      32.8k22452




      32.8k22452










      asked Nov 22 at 4:57









      cmi

      1,000212




      1,000212






















          1 Answer
          1






          active

          oldest

          votes


















          8














          Yep, this works perfectly! You can rigorize this sort of idea by defining some function like



          $$frac{sinleft(x^{10}right)}{x^2}$$



          (where the exponent of $10$ is simply to make sure our function oscillates fast enough).






          share|cite|improve this answer

















          • 1




            Actually simply $sin (x^2)$ suffices.
            – Szeto
            Nov 22 at 5:24










          • @Szeto I imagined lower exponents sufficed, but I didn't want to do out the calculations :-)
            – Carl Schildkraut
            Nov 22 at 5:47






          • 1




            Well, this function is in my memory because of the Fresnel’s integral, which is the first integral I see that converges while its integrand does not vanish. :)
            – Szeto
            Nov 22 at 5:49











          Your Answer





          StackExchange.ifUsing("editor", function () {
          return StackExchange.using("mathjaxEditing", function () {
          StackExchange.MarkdownEditor.creationCallbacks.add(function (editor, postfix) {
          StackExchange.mathjaxEditing.prepareWmdForMathJax(editor, postfix, [["$", "$"], ["\\(","\\)"]]);
          });
          });
          }, "mathjax-editing");

          StackExchange.ready(function() {
          var channelOptions = {
          tags: "".split(" "),
          id: "69"
          };
          initTagRenderer("".split(" "), "".split(" "), channelOptions);

          StackExchange.using("externalEditor", function() {
          // Have to fire editor after snippets, if snippets enabled
          if (StackExchange.settings.snippets.snippetsEnabled) {
          StackExchange.using("snippets", function() {
          createEditor();
          });
          }
          else {
          createEditor();
          }
          });

          function createEditor() {
          StackExchange.prepareEditor({
          heartbeatType: 'answer',
          autoActivateHeartbeat: false,
          convertImagesToLinks: true,
          noModals: true,
          showLowRepImageUploadWarning: true,
          reputationToPostImages: 10,
          bindNavPrevention: true,
          postfix: "",
          imageUploader: {
          brandingHtml: "Powered by u003ca class="icon-imgur-white" href="https://imgur.com/"u003eu003c/au003e",
          contentPolicyHtml: "User contributions licensed under u003ca href="https://creativecommons.org/licenses/by-sa/3.0/"u003ecc by-sa 3.0 with attribution requiredu003c/au003e u003ca href="https://stackoverflow.com/legal/content-policy"u003e(content policy)u003c/au003e",
          allowUrls: true
          },
          noCode: true, onDemand: true,
          discardSelector: ".discard-answer"
          ,immediatelyShowMarkdownHelp:true
          });


          }
          });














          draft saved

          draft discarded


















          StackExchange.ready(
          function () {
          StackExchange.openid.initPostLogin('.new-post-login', 'https%3a%2f%2fmath.stackexchange.com%2fquestions%2f3008755%2fint-0-infty-fx-exists-and-fx-is-differentiable-then-lim-x-to-i%23new-answer', 'question_page');
          }
          );

          Post as a guest















          Required, but never shown

























          1 Answer
          1






          active

          oldest

          votes








          1 Answer
          1






          active

          oldest

          votes









          active

          oldest

          votes






          active

          oldest

          votes









          8














          Yep, this works perfectly! You can rigorize this sort of idea by defining some function like



          $$frac{sinleft(x^{10}right)}{x^2}$$



          (where the exponent of $10$ is simply to make sure our function oscillates fast enough).






          share|cite|improve this answer

















          • 1




            Actually simply $sin (x^2)$ suffices.
            – Szeto
            Nov 22 at 5:24










          • @Szeto I imagined lower exponents sufficed, but I didn't want to do out the calculations :-)
            – Carl Schildkraut
            Nov 22 at 5:47






          • 1




            Well, this function is in my memory because of the Fresnel’s integral, which is the first integral I see that converges while its integrand does not vanish. :)
            – Szeto
            Nov 22 at 5:49
















          8














          Yep, this works perfectly! You can rigorize this sort of idea by defining some function like



          $$frac{sinleft(x^{10}right)}{x^2}$$



          (where the exponent of $10$ is simply to make sure our function oscillates fast enough).






          share|cite|improve this answer

















          • 1




            Actually simply $sin (x^2)$ suffices.
            – Szeto
            Nov 22 at 5:24










          • @Szeto I imagined lower exponents sufficed, but I didn't want to do out the calculations :-)
            – Carl Schildkraut
            Nov 22 at 5:47






          • 1




            Well, this function is in my memory because of the Fresnel’s integral, which is the first integral I see that converges while its integrand does not vanish. :)
            – Szeto
            Nov 22 at 5:49














          8












          8








          8






          Yep, this works perfectly! You can rigorize this sort of idea by defining some function like



          $$frac{sinleft(x^{10}right)}{x^2}$$



          (where the exponent of $10$ is simply to make sure our function oscillates fast enough).






          share|cite|improve this answer












          Yep, this works perfectly! You can rigorize this sort of idea by defining some function like



          $$frac{sinleft(x^{10}right)}{x^2}$$



          (where the exponent of $10$ is simply to make sure our function oscillates fast enough).







          share|cite|improve this answer












          share|cite|improve this answer



          share|cite|improve this answer










          answered Nov 22 at 5:02









          Carl Schildkraut

          11.1k11441




          11.1k11441








          • 1




            Actually simply $sin (x^2)$ suffices.
            – Szeto
            Nov 22 at 5:24










          • @Szeto I imagined lower exponents sufficed, but I didn't want to do out the calculations :-)
            – Carl Schildkraut
            Nov 22 at 5:47






          • 1




            Well, this function is in my memory because of the Fresnel’s integral, which is the first integral I see that converges while its integrand does not vanish. :)
            – Szeto
            Nov 22 at 5:49














          • 1




            Actually simply $sin (x^2)$ suffices.
            – Szeto
            Nov 22 at 5:24










          • @Szeto I imagined lower exponents sufficed, but I didn't want to do out the calculations :-)
            – Carl Schildkraut
            Nov 22 at 5:47






          • 1




            Well, this function is in my memory because of the Fresnel’s integral, which is the first integral I see that converges while its integrand does not vanish. :)
            – Szeto
            Nov 22 at 5:49








          1




          1




          Actually simply $sin (x^2)$ suffices.
          – Szeto
          Nov 22 at 5:24




          Actually simply $sin (x^2)$ suffices.
          – Szeto
          Nov 22 at 5:24












          @Szeto I imagined lower exponents sufficed, but I didn't want to do out the calculations :-)
          – Carl Schildkraut
          Nov 22 at 5:47




          @Szeto I imagined lower exponents sufficed, but I didn't want to do out the calculations :-)
          – Carl Schildkraut
          Nov 22 at 5:47




          1




          1




          Well, this function is in my memory because of the Fresnel’s integral, which is the first integral I see that converges while its integrand does not vanish. :)
          – Szeto
          Nov 22 at 5:49




          Well, this function is in my memory because of the Fresnel’s integral, which is the first integral I see that converges while its integrand does not vanish. :)
          – Szeto
          Nov 22 at 5:49


















          draft saved

          draft discarded




















































          Thanks for contributing an answer to Mathematics Stack Exchange!


          • Please be sure to answer the question. Provide details and share your research!

          But avoid



          • Asking for help, clarification, or responding to other answers.

          • Making statements based on opinion; back them up with references or personal experience.


          Use MathJax to format equations. MathJax reference.


          To learn more, see our tips on writing great answers.





          Some of your past answers have not been well-received, and you're in danger of being blocked from answering.


          Please pay close attention to the following guidance:


          • Please be sure to answer the question. Provide details and share your research!

          But avoid



          • Asking for help, clarification, or responding to other answers.

          • Making statements based on opinion; back them up with references or personal experience.


          To learn more, see our tips on writing great answers.




          draft saved


          draft discarded














          StackExchange.ready(
          function () {
          StackExchange.openid.initPostLogin('.new-post-login', 'https%3a%2f%2fmath.stackexchange.com%2fquestions%2f3008755%2fint-0-infty-fx-exists-and-fx-is-differentiable-then-lim-x-to-i%23new-answer', 'question_page');
          }
          );

          Post as a guest















          Required, but never shown





















































          Required, but never shown














          Required, but never shown












          Required, but never shown







          Required, but never shown

































          Required, but never shown














          Required, but never shown












          Required, but never shown







          Required, but never shown







          Popular posts from this blog

          Plaza Victoria

          In PowerPoint, is there a keyboard shortcut for bulleted / numbered list?

          How to put 3 figures in Latex with 2 figures side by side and 1 below these side by side images but in...